Đến nội dung

Hình ảnh

Toán tính tổng,tích Đại Số- Tuyển tập sưu tầm các bài toán từ Mathlinks.ro

- - - - - tuyển tập-sưu tầm.

  • Please log in to reply
Chủ đề này có 67 trả lời

#21
hxthanh

hxthanh

    Tín đồ $\sum$

  • Hiệp sỹ
  • 3915 Bài viết

Lời giải Bài 10: (Cách khác)

Cách này mang tính "tự nhiên" hơn

Ta có:

$\quad S=\sum_{k=1}^n\dfrac{k+2}{k(k+1)(k+3)}$

$\Rightarrow 3S=\sum_{k=1}^n\dfrac{(2k+6)+k}{k(k+1)(k+3)}$

$\Rightarrow 3S=\sum_{k=1}^n\left(\dfrac{2}{k(k+1)}+\dfrac{1}{(k+1)(k+3)}\right)$

$\Rightarrow 6S=\sum_{k=1}^n\left(\dfrac{4}{k}-\dfrac{4}{k+1}+\dfrac{1}{k+1}-\dfrac{1}{k+2}+\dfrac{1}{k+2}-\dfrac{1}{k+3}\right)$

$\Rightarrow 6S=-\sum_{k=1}^n\Delta\left(\dfrac{4}{k}+\dfrac{1}{k+1}+\dfrac{1}{k+2}\right)$

$\Rightarrow 6S=-\left.\left(\dfrac{4}{k}+\dfrac{1}{k+1}+\dfrac{1}{k+2}\right)\right|_{k=1}^{n+1}$

 

Từ đây ta có điều phải chứng minh :D



#22
dark templar

dark templar

    Kael-Invoker

  • Hiệp sỹ
  • 3788 Bài viết

Spoiler

Đề mới :

 

Bài toán 12: Tính tổng $\sum\limits_{n = 1}^\infty  {\frac{{{F_n}}}{{n!}}} $ trong đó $F_{n}$ là số Fibonacci thứ $n$.

 

Bài toán 13: Cho $m \in \mathbb{Z^+}$.Hãy tính $1.2....m + 2.3....(m + 1) + .... + n(n + 1)...(n + m - 1)$


"Do you still... believe in me ?" Sarah Kerrigan asked Jim Raynor - Starcraft II:Heart Of The Swarm.

#23
hxthanh

hxthanh

    Tín đồ $\sum$

  • Hiệp sỹ
  • 3915 Bài viết

Spoiler

Đề mới :

 

Bài toán 12: Tính tổng $\sum\limits_{n = 1}^\infty  {\frac{{{F_n}}}{{n!}}} $ trong đó $F_{n}$ là số Fibonacci thứ $n$.

 

Bài toán 13: Cho $m \in \mathbb{Z^+}$.Hãy tính $1.2....m + 2.3....(m + 1) + .... + n(n + 1)...(n + m - 1)$

Làm "tạm" bài số 13, vì đây là dạng cơ bản của "nguyên phân" (phép tính ngược của sai phân)

 

Ký hiệu lũy thừa giảm: $x^{\underline n}=\underbrace{x(x-1)...(x-n+1)}_{n\text{ thừa số}}$

Ta có, tổng cần tính là:

$S=\sum_{k=1}^n (k+m-1)^{\underline m}$

 

"Nguyên phân" của $(k+m-1)^{\underline m}$ là $\dfrac{(k+m-1)^{\underline{m+1}}}{m+1}$ giống như kiểu nguyên hàm của $(k+m-1)^m$ vậy!

Các bạn có thể lấy Sai phân để kiểm tra!

Như vậy

$\begin{align*}S&=\dfrac{1}{m+1}\sum_{k=1}^n\Delta[(k+m-1)^{\underline{m+1}}]\\&=\left.\dfrac{(k+m-1)^{\underline{m+1}}}{m+1}\right|_{k=1}^{n+1}\\ &=\dfrac{(n+m)^{\underline{m+1}}-m^{\underline{m+1}}}{m+1}\\&=\dfrac{(n+m)(n+m-1)...(n+1)n-0}{m+1}\\&=m!{n+m\choose m+1}\end{align*}$



#24
Idie9xx

Idie9xx

    Sĩ quan

  • Thành viên
  • 319 Bài viết

Bài toán 13: Cho $m \in \mathbb{Z^+}$.Hãy tính $1.2....m + 2.3....(m + 1) + .... + n(n + 1)...(n + m - 1)$

Viết lại tổng $S=\sum_{i=m}^{n+m-1} A_{i-m}^{i}$ với $A_{k}^{n}=\dfrac{n!}{(n-k)!}$

Ta có $(m+1) \sum_{i=m}^{n+m-1} A_{i-m}^{i} =\sum_{i=m}^{n+m-1} A_{i-m}^{i}((i+1)-(i-m))$

$=\sum_{i=m}^{n+m-1} (A_{i-m}^{i+1}-A_{i-m-1}^{i})=A_{n-1}^{n+m}$

$\Rightarrow S=\dfrac{A_{n-1}^{n+m}}{m+1}$

Không biết đúng không :D

Chậm chân :(


Bài viết đã được chỉnh sửa nội dung bởi Idie9xx: 07-04-2013 - 19:42

$\large \circ \ast R_f\cdot Q_r\cdot 1080\ast \circ$

#25
hxthanh

hxthanh

    Tín đồ $\sum$

  • Hiệp sỹ
  • 3915 Bài viết

Bài 12 mới thật là "lừa đảo"

 

Ta có: $e^x=\sum_{n=0}^{\infty}\dfrac{x^n}{n!}$

 

Nên ....

 

Đáp số là $\Large\dfrac{1}{\sqrt 5}\left[e^{\frac{1+\sqrt 5}{2}}-e^{\frac{1-\sqrt 5}{2}}\right]$

 

:))



#26
dark templar

dark templar

    Kael-Invoker

  • Hiệp sỹ
  • 3788 Bài viết

Viết lại tổng $S=\sum_{i=m}^{n+m-1} A_{i-m}^{i}$ với $A_{k}^{n}=\dfrac{n!}{(n-k)!}$

Ta có $(m+1) \sum_{i=m}^{n+m-1} A_{i-m}^{i} =\sum_{i=m}^{n+m-1} A_{i-m}^{i}((i+1)-(i-m))$

$=\sum_{i=m}^{n+m-1} (A_{i-m}^{i+1}-A_{i-m-1}^{i})=A_{n-1}^{n+m}$

$\Rightarrow S=\dfrac{A_{n-1}^{n+m}}{m+1}$

Không biết đúng không :D

Chậm chân :(

Kết quả này không đúng lắm,phải ra là $\frac{1}{m+1}A_{m+1}^{m+n}$. :)

 

Hic,chém nhanh quá,hôm nay  ngoại lệ cho thêm 2 bài vậy :)

 

Bài toán 14: Tính$S= \sum\limits_{k = 0}^{\left[ {\frac{n}{2}} \right]} {\left( \begin{array}{c}n - k\\k\end{array} \right)} {\left( {\frac{{ - 1}}{4}} \right)^k}$

 

Bài toán 15: Tính $\prod\limits_{k = 1}^{2n} {\tan } \frac{{k\pi }}{{2n + 1}}$ và $\prod\limits_{k = 1}^{n} {\tan } \frac{{k\pi }}{{2n + 1}}$

 

________________________________

hxthanh@to dark: Ngoài Việt Nam mình ra ở nước ngoài người ta có kí hiệu "chỉnh hợp" là $A_n^k$ không em? :)

 

Hai bài này thì thôi, anh nhường các mem khác! :))

 

@Dark templar: Nếu em nhớ không nhầm thì ký hiệu chỉnh hợp này bên nước ngoài ít xài lắm ạ,chủ yếu biểu diễn qua tổ hợp :)


Bài viết đã được chỉnh sửa nội dung bởi dark templar: 07-04-2013 - 20:29

"Do you still... believe in me ?" Sarah Kerrigan asked Jim Raynor - Starcraft II:Heart Of The Swarm.

#27
Idie9xx

Idie9xx

    Sĩ quan

  • Thành viên
  • 319 Bài viết

Kết quả này không đúng lắm,phải ra là $\frac{1}{m+1}A_{m+1}^{m+n}$. :)

Đúng mà. Giống với kết quả của hxthanh :)

Bạn thử đọc kĩ bài của mình xem. Ý tưởng từ bài tính tổng với $m=2$ :D


$\large \circ \ast R_f\cdot Q_r\cdot 1080\ast \circ$

#28
hxthanh

hxthanh

    Tín đồ $\sum$

  • Hiệp sỹ
  • 3915 Bài viết

Bài toán 14: Tính$S= \sum\limits_{k = 0}^{\left[ {\frac{n}{2}} \right]} {\left( \begin{array}{c}n - k\\k\end{array} \right)} {\left( {\frac{{ - 1}}{4}} \right)^k}$

Spoiler

Lời giải Bài toán 14 (Theo phương pháp "Dãy số hóa" - hxthanh)

Spoiler

Đặt $S_n=\sum_{k=0}^{\left\lfloor\frac{n}{2}\right\rfloor}\left(\frac{-1}{4}\right)^k{n-k\choose k}$

Ta có:

$S_{2n}=\sum_{k=0}^n\left(\frac{-1}{4}\right)^k{2n-k\choose k}$

$S_{2n+1}=\sum_{k=0}^n\left(\frac{-1}{4}\right)^k{2n+1-k\choose k}$

Suy ra:

$\begin{align*}S_{2n+1}-S_{2n}&=\left(1+\sum_{k=1}^n\left(\frac{-1}{4}\right)^k{2n+1-k\choose k}\right)-\left(1+\sum_{k=1}^n\left(\frac{-1}{4}\right)^k{2n-k\choose k}\right)\\&=\sum_{k=1}^n\left(\frac{-1}{4}\right)^k{2n-k\choose k-1}\\&=-\dfrac{1}{4}\sum_{k=0}^{n-1}\left(\frac{-1}{4}\right)^k{2n-1-k\choose k}\\&=-\dfrac{1}{4}S_{2n-1}\end{align*}$

Và...

$\begin{align*}S_{2n+2}&=\sum_{k=0}^{n+1}\left(\frac{-1}{4}\right)^k{2n+2-k\choose k}\\&=\sum_{k=0}^{n+1}\left(\frac{-1}{4}\right)^k\left[{2n+1-k\choose k}+{2n+1-k\choose k-1}\right]\\&=\sum_{k=0}^{n+1}\left(\frac{-1}{4}\right)^k{2n+1-k\choose k}+\sum_{k=1}^{n+1}\left(\frac{-1}{4}\right)^k{2n+1-k\choose k-1}\\&=\sum_{k=0}^{n}\left(\frac{-1}{4}\right)^k{2n+1-k\choose k}-\dfrac{1}{4}\sum_{k=0}^{n}\left(\frac{-1}{4}\right)^k{2n-k\choose k}\\&=S_{2n+1}-\dfrac{1}{4}S_{2n}\end{align*}$

Từ hai điều trên suy ra:

$S_n-S_{n-1}=-\dfrac{1}{4}S_{n-2}$

Hay $S_n-\dfrac{1}{2}S_{n-1}=\dfrac{1}{2}\left(S_{n-1}-\dfrac{1}{2}S_{n-2}\right)=...=\dfrac{1}{2^{n-1}}\left(S_1-\dfrac{1}{2}S_0\right)$

Dễ dàng tính được $S_0=S_1=1$, nên ta có:

$\begin{align*}&S_n&-&\dfrac{1}{2}S_{n-1}&=\dfrac{1}{2^n}\\ \dfrac{1}{2}&S_{n-1}&-&\dfrac{1}{2^2}S_{n-2}&=\dfrac{1}{2^n}\\&...&&...&...\\ \dfrac{1}{2^{n-1}}&S_1&-&\dfrac{1}{2^n}S_0&=\dfrac{1}{2^n}\end{align*}$

Cộng tất cả lại ta được:

$$\boxed{\displaystyle S_n=\sum_{k=0}^{\left\lfloor\frac{n}{2}\right\rfloor}\left(\frac{-1}{4}\right)^k{n-k\choose k}=\dfrac{n+1}{2^n}}$$



#29
dark templar

dark templar

    Kael-Invoker

  • Hiệp sỹ
  • 3788 Bài viết

Bài toán 14: Tính$S= \sum\limits_{k = 0}^{\left[ {\frac{n}{2}} \right]} {\left( \begin{array}{c}n - k\\k\end{array} \right)} {\left( {\frac{{ - 1}}{4}} \right)^k}$

Spoiler

Ta có 1 hệ quả của định lý A như sau:

\[\sum\limits_k {\binom{n + ak}{m + bk}{f_k}}  = \left[ {{t^n}} \right]\frac{{{t^m}}}{{{{\left( {1 - t} \right)}^{m + 1}}}}f\left( {\frac{{{t^{b - a}}}}{{{{\left( {1 - t} \right)}^b}}}} \right)\quad \left( {b > a} \right)\]

 

Dễ thấy ${\left( {\frac{{ - 1}}{4}} \right)^k}$ là hệ số của $x^{k}$ trong khai triển $\frac{1}{1+\frac{x}{4}}$ nên:

 

\[\begin{array}{rcl}S &=& \sum\limits_{k = 0}^{\left\lfloor {\frac{n}{2}} \right\rfloor } {{{\left( {\frac{{ - 1}}{4}} \right)}^k}\binom{n - k}{k}} \\&=& \left[ {{t^n}} \right]\frac{1}{{1 - t}}\left[ {\left. {\frac{1}{{1 + \frac{u}{4}}}} \right|u = \frac{{{t^2}}}{{1 - t}}} \right]\\&=& \left[ {{t^n}} \right]\frac{4}{{{{\left( {2 - t} \right)}^2}}} \\&=& 4\left( {n + 1} \right)\left[ {{t^{n + 1}}} \right]\frac{1}{{2 - t}}\quad (*)\\&=& \frac{{4\left( {n + 1} \right)}}{{{2^{n + 2}}}} = \frac{{n + 1}}{{{2^n}}}\end{array}\]
 
Bước $(*)$ này là thực hiện lấy nguyên hàm của khai triển $\frac{1}{(2-t)^2}$
 
@supermember: nhìn mấy lời giải kiểu này thấy vui vui ^^

Bài viết đã được chỉnh sửa nội dung bởi supermember: 12-04-2013 - 19:29

"Do you still... believe in me ?" Sarah Kerrigan asked Jim Raynor - Starcraft II:Heart Of The Swarm.

#30
dark templar

dark templar

    Kael-Invoker

  • Hiệp sỹ
  • 3788 Bài viết


Bài toán 14: Tính$S= \sum\limits_{k = 0}^{\left[ {\frac{n}{2}} \right]} {\left( \begin{array}{c}n - k\\k\end{array} \right)} {\left( {\frac{{ - 1}}{4}} \right)^k}$

 

Bài toán 15: Tính $\prod\limits_{k = 1}^{2n} {\tan } \frac{{k\pi }}{{2n + 1}}$ và $\prod\limits_{k = 1}^{n} {\tan } \frac{{k\pi }}{{2n + 1}}$

 

Lời giải bài toán 15:

Đặt $\tan ((2n + 1)x) = \frac{{x{P_n}(\tan x)}}{{{Q_n}(\tan x)}}$ với ${P_n}(x),{Q_n}(x) \in \mathbb{R}[x]$ và $P_{n}(x)$ là đa thức monic (tức là đa thức có hệ số của số mũ cao nhất bằng 1).

 

Ta có ${P_0}(x) = {Q_0}(x) = 1$ và theo công thức $\tan ((2n + 3)x) = \frac{{\tan ((2n + 1)x) + \tan 2x}}{{1 - \tan ((2n + 1)x)\tan 2x}}$ thì:

 

\[\begin{array}{rcl}{P_{n + 1}}(x) &=& ({x^2} - 1){P_n}(x) - 2{Q_n}(x)\\{Q_{n + 1}}(x) &=& 2{x^2}{P_n}(x) + ({x^2} - 1){Q_n}(x)\end{array}\]
 
Dễ thấy rằng bậc của đa thức $P_{n}$ bằng bậc của đa thức $Q_{n}$ bằng $2n$ và ta cũng có ${Q_n}(0) = {( - 1)^n},{P_n}(0) = (2n + 1){( - 1)^n}$.
 
Vì các nghiệm của đa thức $P_{n}(x)$ là $\left\{ {\tan \frac{{k\pi }}{{2n + 1}}} \right\}_{k = 1}^{2n}$ nên suy ra:

\[\boxed{\displaystyle \prod\limits_{k = 1}^{2n} {\tan } \frac{{k\pi }}{{2n + 1}} = {P_n}(0) = (2n + 1){( - 1)^n}}\]

 

Mặt khác vì $\tan \frac{{k\pi }}{{2n + 1}} =  - \tan \frac{{(2n + 1 - k)\pi }}{{2n + 1}}$  nên suy ra :

\[\boxed{\displaystyle \prod\limits_{k = 1}^n {\tan } \frac{{k\pi }}{{2n + 1}} = \sqrt {2n + 1} }\]

 

 

**********

Đề mới:

 

Bài toán 16: Tính giá trị của tổng vô hạn sau $S=\sum\limits_{r = 1}^{ + \infty } 6 .{\left( {\frac{4}{3}} \right)^{\left\lfloor {\frac{r}{2}} \right\rfloor }}.{\left( {\frac{3}{5}} \right)^{\left\lfloor {\frac{{r - 1}}{2}} \right\rfloor }}$

 

Bài toán 17: Tính tổng $S = \sum\limits_{i = 5}^\infty  {\frac{{2i - 5}}{{\left( {i - 2} \right){i^2}}}} $


"Do you still... believe in me ?" Sarah Kerrigan asked Jim Raynor - Starcraft II:Heart Of The Swarm.

#31
hxthanh

hxthanh

    Tín đồ $\sum$

  • Hiệp sỹ
  • 3915 Bài viết

Bài toán 16: Tính giá trị của tổng vô hạn sau $S=\sum\limits_{r = 1}^{ + \infty } 6 .{\left( {\frac{4}{3}} \right)^{\left\lfloor {\frac{r}{2}} \right\rfloor }}.{\left( {\frac{3}{5}} \right)^{\left\lfloor {\frac{{r - 1}}{2}} \right\rfloor }}$

Ta có:
$\begin{align*}S&=\sum_{1\le r=2k<+\infty}6.\left(\frac{4}{3}\right)^k\left(\frac{3}{5}\right)^{k-1}+\sum_{1\le r=2k-1<+\infty}6.\left(\frac{4}{3}\right)^{k-1}\left(\frac{3}{5}\right)^{k-1}\\&=\sum_{k=1}^{+\infty}8\left(\frac{4}{5}\right)^{k-1}+\sum_{k=1}^{+\infty}6\left(\frac{4}{5}\right)^{k-1}\\&=14\cdot\dfrac{1}{1-\frac{4}{5}}\\&=\boxed{70}\end{align*}$
 

Bài toán 17: Tính tổng $S = \sum\limits_{i = 5}^\infty {\frac{{2i - 5}}{{\left( {i - 2} \right){i^2}}}} $

Ta có:
$S=\sum_{i=5}^\infty \frac{2i-5}{(i-2)i^2}=\sum_{i=1}^\infty \frac{2i+3}{(i+2)(i+4)^2}$
$\quad=\sum_{i=1}^\infty \left(\frac{1}{4(i+4)}-\frac{1}{4(i+2)}+\frac{5}{2(i+4)^2}\right)$
$\quad=\underbrace{\sum_{i=1}^\infty \Delta\left(\frac{1}{4(i+2)}+\frac{1}{4(i+3)}\right)}_{A}+\underbrace{\sum_{i=1}^\infty \frac{5}{2(i+4)^2}}_{B}$
Với:
$A=\lim_{n\to\infty}\sum_{i=1}^n\Delta\left(\frac{1}{4(i+2)}+\frac{1}{4(i+3)}\right)=\lim_{n\to\infty}\left[\frac{1}{4}\left(\frac{1}{n+3}+\frac{1}{n+4}-\frac{1}{3}-\frac{1}{4}\right)\right] = -\frac{7}{48}$
Còn:
$B=-\frac{5}{2}\left(\frac{1}{1^2}+\frac{1}{2^2}+\frac{1}{3^2}+\frac{1}{4^2}\right)+\frac{5}{2}\sum_{i=1}^\infty\frac{1}{i^2}=-\frac{1025}{288}+\frac{5\pi^2}{12}$
 
Do đó: $S=-\frac{7}{48}-\frac{1025}{288}+\frac{5\pi^2}{12}=\boxed{\displaystyle\frac{5\pi^2}{12}-\frac{1067}{288}}$

Spoiler



#32
dark templar

dark templar

    Kael-Invoker

  • Hiệp sỹ
  • 3788 Bài viết

Đề mới:

 

Bài toán 18: Tính 2 tích sau $P_1=\prod_{k=1}^{n}\sin \frac{k\pi}{2n+1}$ và $P_2=\prod_{k=1}^{n}\cos \frac{k\pi}{2n+1}$.

 

Bài toán 19: Tính tổng $S=\sum_{k=1}^{n}\frac{1}{4^{k}\cos^2 \frac{x}{2^{k}}}$.


"Do you still... believe in me ?" Sarah Kerrigan asked Jim Raynor - Starcraft II:Heart Of The Swarm.

#33
hxthanh

hxthanh

    Tín đồ $\sum$

  • Hiệp sỹ
  • 3915 Bài viết

Bài toán 19: Tính tổng $S=\sum_{k=1}^{n}\frac{1}{4^{k}\cos^2 \frac{x}{2^{k}}}$.

Bài 18: Có lẽ cách giải cũng tương tự bài 15

Spoiler

hxthanh xin trình bày lời giải bài 19 như sau:

 

Xét số hạng tổng quát, ta có:

 

$\begin{align*}\dfrac{1}{4^k\cos^2\frac{x}{2^k}}&=\dfrac{\sin^2\frac{x}{2^k}}{4^k\cos^2\frac{x}{2^k}\sin^2\frac{x}{2^k}}\\&=\dfrac{\sin^2\frac{x}{2^k}}{4^{k-1}\sin^2\frac{x}{2^{k-1}}}\\&=\dfrac{4\sin^4\frac{x}{2^k}}{4^k\sin^2\frac{x}{2^k}\sin^2\frac{x}{2^{k-1}}}\\&=\dfrac{4\sin^2\frac{x}{2^k}(1-\cos^2\frac{x}{2^k})}{4^k\sin^2\frac{x}{2^k}\sin^2\frac{x}{2^{k-1}}}\\&=\dfrac{4\sin^2\frac{x}{2^k}-\sin^2\frac{x}{2^{k-1}}}{4^k\sin^2\frac{x}{2^k}\sin^2\frac{x}{2^{k-1}}}\\&=\dfrac{1}{4^{k-1}\sin^2\frac{x}{2^{k-1}}}-\dfrac{1}{4^{k}\sin^2\frac{x}{2^{k}}}\\&=-\Delta\left[\dfrac{1}{4^{k-1}\sin^2\frac{x}{2^{k-1}}}\right]\end{align*}$

 

Do đó theo SPTP, ta có:

$\begin{align*}S&=\left.-\dfrac{1}{4^{k-1}\sin^2\frac{x}{2^{k-1}}}\right|_{k=1}^{n+1}\\&=\boxed{\dfrac{1}{\sin^2x}-\dfrac{1}{4^n\sin^2\frac{x}{2^n}}}\end{align*}$



#34
cool hunter

cool hunter

    Thiếu úy

  • Thành viên
  • 544 Bài viết

Lời giải bài toán 2:
Ta có :
$$\frac{1}{{(i - 1)! + i!}} = \frac{1}{{(i - 1)!(i + 1)}} = \frac{1}{{i!}} - \frac{1}{{(i + 1)!}}$$

Vậy ${a_n} = 1 - \frac{1}{{(n + 1)!}}$.

Do đó :
$$n{a_n} = n - \frac{n}{{(n + 1)!}} = \left( {\frac{{n(n + 1)}}{2} + \frac{1}{{(n + 1)!}}} \right) - \left( {\frac{{(n - 1)n}}{2} + \frac{1}{{n!}}} \right)$$

Từ đó ta tìm được tổng cần tính là ${\frac{{n(n + 1)}}{2} + \frac{1}{{(n + 1)!}} - 1}$

 

Anh cho em hỏi là đoạn tô màu sao anh lại nghĩ ra cách tách thành hiệu như trên vậy?


Thà đừng yêu để giữ mình trong trắng

Lỡ yêu rôì nhất quyết phải thành công

                                                                 


#35
dark templar

dark templar

    Kael-Invoker

  • Hiệp sỹ
  • 3788 Bài viết


Anh cho em hỏi là đoạn tô màu sao anh lại nghĩ ra cách tách thành hiệu như trên vậy?

khúc đó là cách tách đẳng thức thành sai phân quen thuộc. :)

 

Dễ thấy :

\[\frac{n}{{\left( {n + 1} \right)!}} = \frac{{n + 1 - 1}}{{\left( {n + 1} \right)!}} = \frac{1}{{n!}} - \frac{1}{{\left( {n + 1} \right)!}} =  - \Delta \left[ {\frac{1}{{n!}}} \right]\]

 

 

Như vậy ta chỉ cần tách $n$ thành dạng sai phân nữa thôi,tức là cần xác định hàm $f(n)$ sao cho $f(n+1)-f(n)=n$.Ta có thể suy ra được $f(n)$ phải là đa thức bậc 2,tức là $f(n)=an^2+bn(a \neq 0)$ (ta không cần hằng số $c$ nữa vì hiệu giữa $f(n+1)$ và $f(n)$ đã làm triệt tiêu nó ).

 

Bằng phép đồng nhất thức,ta tìm được $f(n)=\frac{n(n-1)}{2}$,hay $n = \Delta \left[ {\frac{{n\left( {n - 1} \right)}}{2}} \right]$.

 

Vậy $\boxed{\displaystyle n - \frac{n}{{\left( {n + 1} \right)!}} = \Delta \left[ {\frac{{n\left( {n - 1} \right)}}{2} + \frac{1}{{n!}}} \right]}$.


"Do you still... believe in me ?" Sarah Kerrigan asked Jim Raynor - Starcraft II:Heart Of The Swarm.

#36
hxthanh

hxthanh

    Tín đồ $\sum$

  • Hiệp sỹ
  • 3915 Bài viết


Bài toán 15: Tính $\prod\limits_{k = 1}^{2n} {\tan } \frac{{k\pi }}{{2n + 1}}$ và $\prod\limits_{k = 1}^{n} {\tan } \frac{{k\pi }}{{2n + 1}}$

Trong lúc chứng minh bổ đề của bài toán này, "tình cờ" xử đẹp bài này :P

Ta có:

$\left[\cos\frac{k\pi}{2n+1}+i\sin\frac{k\pi}{2n+1}\right]^{2n+1}=(-1)^k$

Khai triển nhị thức vế trái:

$\sum_{j=0}^{2n+1}{2n+1\choose j}\left(\cos\frac{k\pi}{2n+1}\right)^{2n+1-j}\left(i\sin\frac{k\pi}{2n+1}\right)^{j}=(-1)^k$

Phần ảo của VT phải bằng $0$ (tương ứng với $j$ lẻ). Suy ra:

$\Rightarrow \sum_{j=0}^{n}{2n+1\choose 2j+1}\left(\cos\frac{k\pi}{2n+1}\right)^{2n-2j}\left(i\sin\frac{k\pi}{2n+1}\right)^{2j+1}=0$

$\Rightarrow \sum_{j=0}^{n}{2n+1\choose 2j+1}(-1)^j\left(\cot^2\frac{k\pi}{2n+1}\right)^{n-j}.i.\left(\sin\frac{k\pi}{2n+1}\right)^{2n+1}=0$

$\Rightarrow \sum_{j=0}^{n}{2n+1\choose 2j+1}(-1)^j\left(\cot^2\frac{k\pi}{2n+1}\right)^{n-j}=0$

Xét phương trình (bậc $n$)

$\sum_{j=0}^{n}{2n+1\choose 2j+1}(-1)^jx^{n-j}=0$

Rõ ràng ta có: $\cot^2\frac{k\pi}{2n+1}$ với $k=1,2,...,n$ là đủ $n$ nghiệm phân biệt của phương trình trên.

Theo định lý Viète, ta có tổng các nghiệm là:

$\sum_{k=1}^n\cot^2\frac{k\pi}{2n+1}=-\dfrac{\displaystyle -{2n+1\choose 3}}{\displaystyle {2n+1\choose 1}}=\frac{n(2n-1)}{3}$

Còn tích các nghiệm là:

$\prod_{k=1}^n\cot^2\frac{k\pi}{2n+1}=(-1)^n\dfrac{\displaystyle (-1)^n{2n+1\choose 2n+1}}{\displaystyle {2n+1\choose 1}}=\frac{1}{2n+1}$

 

Suy ra:

$\boxed{\displaystyle \prod_{k=1}^n \tan\frac{k\pi}{2n+1}=\sqrt{2n+1}}$ Do $\tan x>0, \forall x\in\left(0,\frac{\pi}{2}\right)$

 

Mặt khác: vì $\tan\frac{k\pi}{2n+1}=-\tan\frac{(2n+1-k)\pi}{2n+1}$ nên

 

$\boxed{\displaystyle \prod_{k=1}^{2n} \tan\frac{k\pi}{2n+1}=(-1)^n(2n+1)}$



#37
dark templar

dark templar

    Kael-Invoker

  • Hiệp sỹ
  • 3788 Bài viết

Bài toán 18: Tính 2 tích sau $P_1=\prod_{k=1}^{n}\sin \frac{k\pi}{2n+1}$ và $P_2=\prod_{k=1}^{n}\cos \frac{k\pi}{2n+1}$.

Lời giải bài toán 18:

Trước tiên ta xét tích $P_1=\prod_{k=1}^{n}\sin \frac{k\pi}{2n+1}$

 

Ta dễ thấy rằng PT $x^{2n+1}=1$ có 1 nghiệm thực là $x=1$ và $2n$ nghiệm phức thỏa mãn PT $\sum_{k=0}^{2n}z^{2k}=0 \quad (*)$ và bộ $2n$ nghiệm phức này được xác định bởi công thức $\boxed{\displaystyle {z_k} = \cos \frac{{2k\pi }}{{2n + 1}} + i\sin \frac{{2k\pi }}{{2n + 1}} = {e^{\frac{{2k\pi }}{{2n + 1}}i}}\left( {k = \overline {0;2n} } \right)}$.

 

Từ đó ta có được khai triển sau:

$\begin{array}{l}{x^{2n + 1}} - 1 = \prod\limits_{k = 0}^{2n} {\left( {x - {e^{\frac{{2\pi k}}{{2n + 1}}i}}} \right)} \\\Rightarrow \frac{{{x^{2n + 1}} - 1}}{{x - 1}} = \prod\limits_{k = 1}^{2n} {\left( {x - {e^{\frac{{2\pi k}}{{2n + 1}}i}}} \right)} \\\Rightarrow \mathop {\lim }\limits_{x \to 1} \frac{{{x^{2n + 1}} - 1}}{{x - 1}} = \mathop {\lim }\limits_{x \to 1} \prod\limits_{k = 1}^{2n} {\left( {x - {e^{\frac{{2\pi k}}{{2n + 1}}i}}} \right)} \\\Rightarrow 2n + 1 = \prod\limits_{k = 1}^{2n} {\left( {1 - {e^{\frac{{2\pi k}}{{2n + 1}}i}}} \right)}  = \prod\limits_{k = 1}^n {\left( {1 - {e^{\frac{{2\pi k}}{{2n + 1}}i}}} \right)} \left( {1 - {e^{ - \frac{{2\pi k}}{{2n + 1}}i}}} \right)\\\Rightarrow 2n + 1 = \prod\limits_{k = 1}^n {\left| {1 - {e^{\frac{{2\pi k}}{{2n + 1}}i}}} \right|} \left| {1 - {e^{ - \frac{{2\pi k}}{{2n + 1}}i}}} \right| = \prod\limits_{k = 1}^n {{{\left| {1 - {e^{\frac{{2\pi k}}{{2n + 1}}i}}} \right|}^2}} \\\Rightarrow 2n + 1 = \prod\limits_{k = 1}^n {\left( {{{\left( {1 - \cos \frac{{2\pi k}}{{2n + 1}}} \right)}^2} + {{\sin }^2}\frac{{2\pi k}}{{2n + 1}}} \right)} \\\Rightarrow 2n + 1 = \prod\limits_{k = 1}^n {\left( {{{\left( {2{{\sin }^2}\frac{{\pi k}}{{2n + 1}}} \right)}^2} + 4{{\sin }^2}\frac{{\pi k}}{{2n + 1}}{{\cos }^2}\frac{{\pi k}}{{2n + 1}}} \right)} \\\Rightarrow 2n + 1 = \prod\limits_{k = 1}^n 4 {\sin ^2}\frac{{\pi k}}{{2n + 1}}\\\Rightarrow 2n + 1 = {4^n}{\left( {\prod\limits_{k = 1}^n {\sin } \frac{{\pi k}}{{2n + 1}}} \right)^2}\end{array}$

 

Từ đó suy ra $\boxed{\displaystyle \prod\limits_{k = 1}^n {\sin } \frac{{\pi k}}{{2n + 1}} = \frac{{\sqrt {2n + 1} }}{{{2^n}}}}$.

 

 

Tiếp theo ta cần tính $P_2=\prod_{k=1}^{n}\cos \frac{k\pi}{2n+1}$.

 

Đặt $C = \prod\limits_{k = 1}^n {\cos } \frac{{k\pi }}{{2n + 1}}{\rm{  }}$ và $S = \prod\limits_{k = 1}^n {\sin } \frac{{k\pi }}{{2n + 1}}$.Ta có:

 

$\begin{array}{l}{2^n}CS = \prod\limits_{k = 1}^n 2 \sin \frac{{k\pi }}{{2n + 1}}\cos \frac{{k\pi }}{{2n + 1}}\\\Rightarrow {2^n}CS = \prod\limits_{k = 1}^n {\sin } \frac{{2k\pi }}{{2n + 1}}\\\Rightarrow {2^n}CS = \sin \frac{{2\pi }}{{2n + 1}}\sin \frac{{4\pi }}{{2n + 1}}\sin \frac{{6\pi }}{{2n + 1}} \cdots \sin \frac{{2(n - 1)\pi }}{{2n + 1}}\sin \frac{{2n\pi }}{{2n + 1}}\\\Rightarrow {2^n}CS = \sin \frac{{2\pi }}{{2n + 1}}\sin \frac{{4\pi }}{{2n + 1}}\sin \frac{{6\pi }}{{2n + 1}} \cdots \sin \left( {\pi  - \frac{{2(n - 1)\pi }}{{2n + 1}}} \right)\sin \left( {\pi  - \frac{{2n\pi }}{{2n + 1}}} \right)\\\Rightarrow {2^n}CS = \sin \frac{{2\pi }}{{2n + 1}}\sin \frac{{4\pi }}{{2n + 1}}\sin \frac{{6\pi }}{{2n + 1}} \cdots \sin \frac{{3\pi }}{{2n + 1}}\sin \frac{\pi }{{2n + 1}}\\\Rightarrow {2^n}CS = S\end{array}$
 
Từ đó suy ra $C=\frac{1}{2^{n}} \quad \text{(do $S \neq 0$)}$,hay $\boxed{\displaystyle \prod\limits_{k = 1}^n {\cos } \frac{{k\pi }}{{2n + 1}}=\frac{1}{2^{n}}}$.

 

**********

Đề mới:

 

Bài toán 20: Tính tổng $S = sec\frac{\pi }{{13}} + sec\frac{{3\pi }}{{13}} + sec\frac{{5\pi }}{{13}} + sec\frac{{7\pi }}{{13}} + sec\frac{{9\pi }}{{13}} + sec\frac{{11\pi }}{{13}}$.

 

Trong đó ký hiệu $\sec x=\frac{1}{\cos x}$.

 

Hãy tổng quát hóa bài toán.

 

Bài toán 21: Tính tổng ${S_n} = \sum\limits_{k = 1}^n {\frac{{{{( - 1)}^{k - 1}}}}{{(n - k)!(n + k)!}}} $.


Bài viết đã được chỉnh sửa nội dung bởi dark templar: 21-04-2013 - 11:05

"Do you still... believe in me ?" Sarah Kerrigan asked Jim Raynor - Starcraft II:Heart Of The Swarm.

#38
hxthanh

hxthanh

    Tín đồ $\sum$

  • Hiệp sỹ
  • 3915 Bài viết


Bài toán 21: Tính tổng ${S_n} = \sum\limits_{k = 1}^n {\frac{{{{( - 1)}^{k - 1}}}}{{(n - k)!(n + k)!}}} $.

Tư tưởng bài này không khác gì bài 5

 

$\dfrac{(-1)^{k-1}}{(n+k)!(n-k)!}=\dfrac{1}{2n}\dfrac{(-1)^{k-1}\big((n+k)+(n-k)\big)}{(n+k)!(n-k)!}$

$\quad = \dfrac{1}{2n}\left[\dfrac{(-1)^{k-1}}{(n+k-1)!(n-k)!}-\dfrac{(-1)^k}{(n+k)!(n-k-1)!}\right]$

$\quad =-\dfrac{1}{2n}\Delta\left[\dfrac{(-1)^{k-1}}{(n+k-1)!(n-k)!}\right]$

 

Do đó:

$\begin{align*}S_n&=\dfrac{(-1)^{n-1}}{(2n)!}+\sum_{k=1}^{n-1}\dfrac{(-1)^{k-1}}{(n+k)!(n-k)!}\\&=\dfrac{(-1)^{n-1}}{(2n)!}+\left.\dfrac{(-1)^k}{2n(n+k-1)!(n-k)!}\right|_{k=1}^{n}\\&=\dfrac{(-1)^{n-1}}{(2n)!}+\dfrac{(-1)^{n}}{(2n)(2n-1)!}+\dfrac{1}{2n(n-1)!n!}\\&=\boxed{ \dfrac{1}{2(n!)^2}}\end{align*}$



#39
nthoangcute

nthoangcute

    Thiếu tá

  • Thành viên
  • 2003 Bài viết

Bài toán 20: Tính tổng $S = sec\frac{\pi }{{13}} + sec\frac{{3\pi }}{{13}} + sec\frac{{5\pi }}{{13}} + sec\frac{{7\pi }}{{13}} + sec\frac{{9\pi }}{{13}} + sec\frac{{11\pi }}{{13}}$.

Trong đó ký hiệu $\sec x=\frac{1}{\cos x}$.

Hãy tổng quát hóa bài toán.

 

Em định tổng quát như này, liệu có đẹp ?

Bài toán 20*: Chứng minh: $$\sum^n_{k=1} \sec \frac{(2k-1) \pi}{2n+1} = \frac{1}{2}- \left ( n+\frac{1}{2} \right ) \left ( -1 \right )^n$$


Bài viết đã được chỉnh sửa nội dung bởi nthoangcute: 21-04-2013 - 11:55

BÙI THẾ VIỆT - Chuyên gia Thủ Thuật CASIO

 

Facebook : facebook.com/viet.alexander.7


Youtube : youtube.com/nthoangcute


Gmail : [email protected]


SÐT : 0965734893


#40
dark templar

dark templar

    Kael-Invoker

  • Hiệp sỹ
  • 3788 Bài viết


Bài toán 20: Tính tổng $S = sec\frac{\pi }{{13}} + sec\frac{{3\pi }}{{13}} + sec\frac{{5\pi }}{{13}} + sec\frac{{7\pi }}{{13}} + sec\frac{{9\pi }}{{13}} + sec\frac{{11\pi }}{{13}}$.

 

Trong đó ký hiệu $\sec x=\frac{1}{\cos x}$.

 

Hãy tổng quát hóa bài toán.

Lời giải bài toán 20:

Đặt ${z_k} = \cos \frac{{(2k - 1)\pi }}{{13}} + {\rm{i}}{\rm{.}}\sin \frac{{({\rm{2k - 1}})\pi }}{{{\rm{13}}}}$ với ${\rm{k = 1}},{\rm{2}},...,{\rm{13}}$ là các nghiệm phức của PT :

${{\rm{z}}^{{\rm{13}}}}{\rm{ + 1 = }}({\rm{z + 1}}).({{\rm{z}}^{{\rm{12}}}}{\rm{ - }}{{\rm{z}}^{{\rm{11}}}}{\rm{ + }}{{\rm{z}}^{{\rm{10}}}}{\rm{ - }}{{\rm{z}}^{\rm{9}}}{\rm{ + }}{{\rm{z}}^{\rm{8}}}{\rm{ - }}{{\rm{z}}^{\rm{7}}}{\rm{ + }}{{\rm{z}}^{\rm{6}}}{\rm{ - }}{{\rm{z}}^{\rm{5}}}{\rm{ + }}{{\rm{z}}^{\rm{4}}}{\rm{ - }}{{\rm{z}}^{\rm{3}}}{\rm{ + }}{{\rm{z}}^{\rm{2}}}{\rm{ - z + 1}}){\rm{ = 0}}$

$ \Leftrightarrow (z + 1).{z^6}.\left( {{z^6} - {z^5} + {z^4} - {z^3} + {z^2} - z + 1 - \frac{1}{z} + \frac{1}{{{z^2}}} - \frac{1}{{{z^3}}} + \frac{1}{{{z^4}}} - \frac{1}{{{z^5}}} + \frac{1}{{{z^6}}}} \right) = 0$

 

Đặt ${z_k} + \frac{1}{{{z_k}}} = 2\cos \frac{{(2k - 1)\pi }}{{13}} = {x_k}$,với $k = 1,2,...,6$ thì:

$z + \frac{1}{z} = x$

${z^2} + \frac{1}{{{z^2}}} = {\left( {z + \frac{1}{z}} \right)^2} - 2 = {x^2} - 2$

${z^3} + \frac{1}{{{z^3}}} = {\left( {z + \frac{1}{z}} \right)^3} - 3\left( {z + \frac{1}{z}} \right) = {x^3} - 3x$

${z^4} + \frac{1}{{{z^4}}} = {\left( {z + \frac{1}{z}} \right)^4} - 4\left( {{z^2} + \frac{1}{{{z^2}}}} \right) - 6 = {x^4} - 4({x^2} - 2) - 6 = {x^4} - 4{x^2} + 2$

${z^5} + \frac{1}{{{z^5}}} = {\left( {z + \frac{1}{z}} \right)^5} - 5\left( {{z^3} + \frac{1}{{{z^3}}}} \right) - 10\left( {z + \frac{1}{z}} \right) = {x^5} - 5({x^3} - 3x) - 10x = {x^5} - 5{x^3} + 5x$

 

$\begin{array}{l}{z^6} + \frac{1}{{{z^6}}} = {\left( {z + \frac{1}{z}} \right)^6} - 6\left( {{z^4} + \frac{1}{{{z^4}}}} \right) - 15\left( {{z^2} + \frac{1}{{{z^2}}}} \right) - 20\\= {x^6} - 6[{x^4} - 4({x^2} - 2) - 6] - 15({x^2} - 2) - 20 = {x^6} - 6{x^4} + 9{x^2} - 2\end{array}$
 
Vậy thì:
${z^6} - {z^5} + {z^4} - {z^3} + {z^2} - z + 1 - \frac{1}{z} + \frac{1}{{{z^2}}} - \frac{1}{{{z^3}}} + \frac{1}{{{z^4}}} - \frac{1}{{{z^5}}} + \frac{1}{{{z^6}}}$

$ = ({x^6} - 6{x^4} + 9{x^2} - 2) - ({x^5} - 5{x^3} + 5x) + ({x^4} - 4{x^2} + 2) - ({x^3} - 3x) + ({x^2} - 2) - x + 1$

$ = {x^6} - {x^5} - 5{x^4} + 4{x^3} + 6{x^2} - 3x - 1 = \sum\limits_{k = 0}^6 {{a_k}} {x^k} = 0$

 

Từ đó ta tính được :

$$\begin{array}{rcl}S &=& \sum\limits_{k = 1}^6 {\sec } \frac{{(2k - 1)\pi }}{{13}}\\&=& \frac{{\sum\limits_{j = 1}^6 {{2^{ - 5}}} \prod\limits_{k = 1,k \ne j}^6 {{x_k}} }}{{{2^{ - 6}}\prod\limits_{k = 1}^6 {{x_k}} }}\\&=& \frac{{2\cdot( - {a_1})}}{{{a_0}}} = \frac{{2\cdot3}}{{ - 1}} = \boxed{\displaystyle - 6}\end{array}$$

 

**********

Đề mới:

 

Bài toán 22: Cho ${a_k} = 1 + \frac{1}{3} + \frac{1}{5} + ... + \frac{1}{{2k - 1}}$ với ${\rm{k}} = {\rm{1}},{\rm{2}},...,{\rm{n}}$

Tính tổng $\frac{1}{2}a_n^2 + {({a_n} - {a_1})^2} + {({a_n} - {a_2})^2} + ... + {({a_n} - {a_{n - 1}})^2}$

 

Bài toán 23: Tính tổng $\sum\limits_{k = 0}^{n - 1} {{{\cos }^{ - 1}}} \left( {\frac{{{n^2} + {k^2} + k}}{{\sqrt {{n^4} + {k^4} + 2{k^3} + 2{n^2}{k^2} + 2{n^2}k + {n^2} + {k^2}} }}} \right)$


"Do you still... believe in me ?" Sarah Kerrigan asked Jim Raynor - Starcraft II:Heart Of The Swarm.





Được gắn nhãn với một hoặc nhiều trong số những từ khóa sau: tuyển tập-sưu tầm.

0 người đang xem chủ đề

0 thành viên, 0 khách, 0 thành viên ẩn danh